If at each point of a closed interval the $m$-th derivative of $f$ is $0$ for $m$ large enough, then $f$ is...












2












$begingroup$


I am working on exercice 9.5.2 of Analysis by Zorich and I am stuck at the question b.




a) A set $Esubset X$ of a metric space $(X,d)$ is nowhere dense in X if it is not dense in any ball, that is, if for every ball $B(x,r)$ there is a second ball $B(x_1,r_1)subset B(x,r)$ containing no points of the set $E$.
A set E is of first category in X if it can be represented as a countable union of nowhere dense sets. A set that is not of first category is of second category in $X$. Show that a complete metric space is set of second category (in itself).



b) Show that if a function $fin C^{(infty)}[a,b]$ is such that $forall xin [a,b] ;exists nin mathbb{N} ;forall m>n ;(f^{(m)}(x)=0)$, then $f$ is a polynomial.




Here's my try: Define the sets $S_n:={xin[a,b]mid f^{(m)}(x)=0; forall m>n}$. Then $cup_{n=1}^{infty} S_n =[a,b]$. As $[a,b]$ is a complete metric space, $S_n$ cannot be all nowhere dense. Define $Y:={xin [a,b] mid text{ there exists a neighborhood of } x text{ and } n text{ such that } S_n text{ is dense in that neighborhood}$. I want to say that $Y=[a,b]$ so that I could conclude with the compactness of $[a,b]$. But all I can say is that the complement of $Y$ in $[a,b]$ contains no interval.



Can you help me? Thanks!










share|cite|improve this question









$endgroup$

















    2












    $begingroup$


    I am working on exercice 9.5.2 of Analysis by Zorich and I am stuck at the question b.




    a) A set $Esubset X$ of a metric space $(X,d)$ is nowhere dense in X if it is not dense in any ball, that is, if for every ball $B(x,r)$ there is a second ball $B(x_1,r_1)subset B(x,r)$ containing no points of the set $E$.
    A set E is of first category in X if it can be represented as a countable union of nowhere dense sets. A set that is not of first category is of second category in $X$. Show that a complete metric space is set of second category (in itself).



    b) Show that if a function $fin C^{(infty)}[a,b]$ is such that $forall xin [a,b] ;exists nin mathbb{N} ;forall m>n ;(f^{(m)}(x)=0)$, then $f$ is a polynomial.




    Here's my try: Define the sets $S_n:={xin[a,b]mid f^{(m)}(x)=0; forall m>n}$. Then $cup_{n=1}^{infty} S_n =[a,b]$. As $[a,b]$ is a complete metric space, $S_n$ cannot be all nowhere dense. Define $Y:={xin [a,b] mid text{ there exists a neighborhood of } x text{ and } n text{ such that } S_n text{ is dense in that neighborhood}$. I want to say that $Y=[a,b]$ so that I could conclude with the compactness of $[a,b]$. But all I can say is that the complement of $Y$ in $[a,b]$ contains no interval.



    Can you help me? Thanks!










    share|cite|improve this question









    $endgroup$















      2












      2








      2


      1



      $begingroup$


      I am working on exercice 9.5.2 of Analysis by Zorich and I am stuck at the question b.




      a) A set $Esubset X$ of a metric space $(X,d)$ is nowhere dense in X if it is not dense in any ball, that is, if for every ball $B(x,r)$ there is a second ball $B(x_1,r_1)subset B(x,r)$ containing no points of the set $E$.
      A set E is of first category in X if it can be represented as a countable union of nowhere dense sets. A set that is not of first category is of second category in $X$. Show that a complete metric space is set of second category (in itself).



      b) Show that if a function $fin C^{(infty)}[a,b]$ is such that $forall xin [a,b] ;exists nin mathbb{N} ;forall m>n ;(f^{(m)}(x)=0)$, then $f$ is a polynomial.




      Here's my try: Define the sets $S_n:={xin[a,b]mid f^{(m)}(x)=0; forall m>n}$. Then $cup_{n=1}^{infty} S_n =[a,b]$. As $[a,b]$ is a complete metric space, $S_n$ cannot be all nowhere dense. Define $Y:={xin [a,b] mid text{ there exists a neighborhood of } x text{ and } n text{ such that } S_n text{ is dense in that neighborhood}$. I want to say that $Y=[a,b]$ so that I could conclude with the compactness of $[a,b]$. But all I can say is that the complement of $Y$ in $[a,b]$ contains no interval.



      Can you help me? Thanks!










      share|cite|improve this question









      $endgroup$




      I am working on exercice 9.5.2 of Analysis by Zorich and I am stuck at the question b.




      a) A set $Esubset X$ of a metric space $(X,d)$ is nowhere dense in X if it is not dense in any ball, that is, if for every ball $B(x,r)$ there is a second ball $B(x_1,r_1)subset B(x,r)$ containing no points of the set $E$.
      A set E is of first category in X if it can be represented as a countable union of nowhere dense sets. A set that is not of first category is of second category in $X$. Show that a complete metric space is set of second category (in itself).



      b) Show that if a function $fin C^{(infty)}[a,b]$ is such that $forall xin [a,b] ;exists nin mathbb{N} ;forall m>n ;(f^{(m)}(x)=0)$, then $f$ is a polynomial.




      Here's my try: Define the sets $S_n:={xin[a,b]mid f^{(m)}(x)=0; forall m>n}$. Then $cup_{n=1}^{infty} S_n =[a,b]$. As $[a,b]$ is a complete metric space, $S_n$ cannot be all nowhere dense. Define $Y:={xin [a,b] mid text{ there exists a neighborhood of } x text{ and } n text{ such that } S_n text{ is dense in that neighborhood}$. I want to say that $Y=[a,b]$ so that I could conclude with the compactness of $[a,b]$. But all I can say is that the complement of $Y$ in $[a,b]$ contains no interval.



      Can you help me? Thanks!







      analysis






      share|cite|improve this question













      share|cite|improve this question











      share|cite|improve this question




      share|cite|improve this question










      asked Dec 16 '18 at 15:26









      JiuJiu

      515113




      515113






















          3 Answers
          3






          active

          oldest

          votes


















          1












          $begingroup$

          Here is a sketch of a possible approach, which may have some holes, but it is too long for a comment. Maybe someone can patch it up.Picking up on your idea, set



          $T = {tin [a,b]: forall (c,d)ni t: frestriction_{(c,d)}$ is not a polynomial$}$



          Then $T$ is non-empty and closed. By construction, it has no isolated points. Now we may apply the Baire theorem on ${Tcap S_n}$ to find an interval $(c,d)$ such that for some $nin mathbb N, (c,d)cap Tsubset S_n$.



          Now, $f$ is a polynomial on $(c,d)setminus T,$ which is open and so contains an interval $(alpha,beta)$, which we may take to be maximal: indeed, $(c,d)setminus T$ is a countable disjoint union of intervals $bigcup_n(a_n,b_n)$ and so ${x:a',b'in (a,b)setminus T text{and} b'-a'ge b_n-a_n}$ is a maximal interval in $(a,b)setminus T$.



          Now, either $alpha$ or $betain T$. Suppose $alphain T$. Then, on every interval $(c',c'')$ containing $alpha, f$ is $not$ a polynomial. Choose $c'''$ such that $c'<alpha<c'''<c''<d$. Then, $f$ is not a polynomial on $(c''',c'').$ But since $(c''',c'')subseteq (alpha,beta)$, so we have a contradiction.



          So, either there is no such interval $(c,d)$ or $T$ is empty. In either case, the result follows.






          share|cite|improve this answer











          $endgroup$













          • $begingroup$
            Thank you for your answer. I’ll have to think about it for a while.
            $endgroup$
            – Jiu
            Dec 17 '18 at 5:09










          • $begingroup$
            I think my proof needs work and I am hoping somebody can supply the missing details/fix the errors.
            $endgroup$
            – Matematleta
            Dec 17 '18 at 21:12



















          0












          $begingroup$

          You can ‘trace’ the $m$th derivative back to the original function recursively. If $f^{(m)}(x) = 0$ for all $x in [a,b]$, then $f^{(m-1)}(x) = int f^{(m)}(x) , mathrm{d}x = int 0,mathrm{d}x=c_0$. Subsequently, $f^{(m-2)}(x) = int c_0,mathrm{d}x=c_1x+c_0$. Applying this technique $m$ times, we see that $f(x) = c_mx^m + c_{m-1}x^{m-1}+dots + c_1x + c_0$, which is a polynomial by definition.



          (Note that the indices $c_1, c_0$ etc. are not necessarily the same for each iteration, they are just arbitrary constants, where the index indicates the power of $x$ associated to it.)






          share|cite|improve this answer









          $endgroup$













          • $begingroup$
            Thanks for your answer. But the assumption is not that there is an m uniformly for all x, but for all x m-th derivative is zero for m large enough.
            $endgroup$
            – Jiu
            Dec 19 '18 at 13:36










          • $begingroup$
            But doesn't that imply there also exists an $m$ for which $f^{(m)}(x)=0$ for all $x$? (That is, the maximum $m$ that you need for any individual $x$.)
            $endgroup$
            – molarmass
            Dec 19 '18 at 13:40










          • $begingroup$
            the supremum maybe infinite
            $endgroup$
            – Jiu
            Dec 19 '18 at 13:40





















          0












          $begingroup$


          b) Show that if a function $fin C^{(infty)}[a,b]$ is such that $forall xin [a,b] ;exists nin mathbb{N} ;forall m>n ;(f^{(m)}(x)=0)$, then $f$ is a polynomial.




          It suffices to require that $f^n(x)=0$ and this is a well-known fact.






          share|cite|improve this answer









          $endgroup$













            Your Answer





            StackExchange.ifUsing("editor", function () {
            return StackExchange.using("mathjaxEditing", function () {
            StackExchange.MarkdownEditor.creationCallbacks.add(function (editor, postfix) {
            StackExchange.mathjaxEditing.prepareWmdForMathJax(editor, postfix, [["$", "$"], ["\\(","\\)"]]);
            });
            });
            }, "mathjax-editing");

            StackExchange.ready(function() {
            var channelOptions = {
            tags: "".split(" "),
            id: "69"
            };
            initTagRenderer("".split(" "), "".split(" "), channelOptions);

            StackExchange.using("externalEditor", function() {
            // Have to fire editor after snippets, if snippets enabled
            if (StackExchange.settings.snippets.snippetsEnabled) {
            StackExchange.using("snippets", function() {
            createEditor();
            });
            }
            else {
            createEditor();
            }
            });

            function createEditor() {
            StackExchange.prepareEditor({
            heartbeatType: 'answer',
            autoActivateHeartbeat: false,
            convertImagesToLinks: true,
            noModals: true,
            showLowRepImageUploadWarning: true,
            reputationToPostImages: 10,
            bindNavPrevention: true,
            postfix: "",
            imageUploader: {
            brandingHtml: "Powered by u003ca class="icon-imgur-white" href="https://imgur.com/"u003eu003c/au003e",
            contentPolicyHtml: "User contributions licensed under u003ca href="https://creativecommons.org/licenses/by-sa/3.0/"u003ecc by-sa 3.0 with attribution requiredu003c/au003e u003ca href="https://stackoverflow.com/legal/content-policy"u003e(content policy)u003c/au003e",
            allowUrls: true
            },
            noCode: true, onDemand: true,
            discardSelector: ".discard-answer"
            ,immediatelyShowMarkdownHelp:true
            });


            }
            });














            draft saved

            draft discarded


















            StackExchange.ready(
            function () {
            StackExchange.openid.initPostLogin('.new-post-login', 'https%3a%2f%2fmath.stackexchange.com%2fquestions%2f3042725%2fif-at-each-point-of-a-closed-interval-the-m-th-derivative-of-f-is-0-for-m%23new-answer', 'question_page');
            }
            );

            Post as a guest















            Required, but never shown

























            3 Answers
            3






            active

            oldest

            votes








            3 Answers
            3






            active

            oldest

            votes









            active

            oldest

            votes






            active

            oldest

            votes









            1












            $begingroup$

            Here is a sketch of a possible approach, which may have some holes, but it is too long for a comment. Maybe someone can patch it up.Picking up on your idea, set



            $T = {tin [a,b]: forall (c,d)ni t: frestriction_{(c,d)}$ is not a polynomial$}$



            Then $T$ is non-empty and closed. By construction, it has no isolated points. Now we may apply the Baire theorem on ${Tcap S_n}$ to find an interval $(c,d)$ such that for some $nin mathbb N, (c,d)cap Tsubset S_n$.



            Now, $f$ is a polynomial on $(c,d)setminus T,$ which is open and so contains an interval $(alpha,beta)$, which we may take to be maximal: indeed, $(c,d)setminus T$ is a countable disjoint union of intervals $bigcup_n(a_n,b_n)$ and so ${x:a',b'in (a,b)setminus T text{and} b'-a'ge b_n-a_n}$ is a maximal interval in $(a,b)setminus T$.



            Now, either $alpha$ or $betain T$. Suppose $alphain T$. Then, on every interval $(c',c'')$ containing $alpha, f$ is $not$ a polynomial. Choose $c'''$ such that $c'<alpha<c'''<c''<d$. Then, $f$ is not a polynomial on $(c''',c'').$ But since $(c''',c'')subseteq (alpha,beta)$, so we have a contradiction.



            So, either there is no such interval $(c,d)$ or $T$ is empty. In either case, the result follows.






            share|cite|improve this answer











            $endgroup$













            • $begingroup$
              Thank you for your answer. I’ll have to think about it for a while.
              $endgroup$
              – Jiu
              Dec 17 '18 at 5:09










            • $begingroup$
              I think my proof needs work and I am hoping somebody can supply the missing details/fix the errors.
              $endgroup$
              – Matematleta
              Dec 17 '18 at 21:12
















            1












            $begingroup$

            Here is a sketch of a possible approach, which may have some holes, but it is too long for a comment. Maybe someone can patch it up.Picking up on your idea, set



            $T = {tin [a,b]: forall (c,d)ni t: frestriction_{(c,d)}$ is not a polynomial$}$



            Then $T$ is non-empty and closed. By construction, it has no isolated points. Now we may apply the Baire theorem on ${Tcap S_n}$ to find an interval $(c,d)$ such that for some $nin mathbb N, (c,d)cap Tsubset S_n$.



            Now, $f$ is a polynomial on $(c,d)setminus T,$ which is open and so contains an interval $(alpha,beta)$, which we may take to be maximal: indeed, $(c,d)setminus T$ is a countable disjoint union of intervals $bigcup_n(a_n,b_n)$ and so ${x:a',b'in (a,b)setminus T text{and} b'-a'ge b_n-a_n}$ is a maximal interval in $(a,b)setminus T$.



            Now, either $alpha$ or $betain T$. Suppose $alphain T$. Then, on every interval $(c',c'')$ containing $alpha, f$ is $not$ a polynomial. Choose $c'''$ such that $c'<alpha<c'''<c''<d$. Then, $f$ is not a polynomial on $(c''',c'').$ But since $(c''',c'')subseteq (alpha,beta)$, so we have a contradiction.



            So, either there is no such interval $(c,d)$ or $T$ is empty. In either case, the result follows.






            share|cite|improve this answer











            $endgroup$













            • $begingroup$
              Thank you for your answer. I’ll have to think about it for a while.
              $endgroup$
              – Jiu
              Dec 17 '18 at 5:09










            • $begingroup$
              I think my proof needs work and I am hoping somebody can supply the missing details/fix the errors.
              $endgroup$
              – Matematleta
              Dec 17 '18 at 21:12














            1












            1








            1





            $begingroup$

            Here is a sketch of a possible approach, which may have some holes, but it is too long for a comment. Maybe someone can patch it up.Picking up on your idea, set



            $T = {tin [a,b]: forall (c,d)ni t: frestriction_{(c,d)}$ is not a polynomial$}$



            Then $T$ is non-empty and closed. By construction, it has no isolated points. Now we may apply the Baire theorem on ${Tcap S_n}$ to find an interval $(c,d)$ such that for some $nin mathbb N, (c,d)cap Tsubset S_n$.



            Now, $f$ is a polynomial on $(c,d)setminus T,$ which is open and so contains an interval $(alpha,beta)$, which we may take to be maximal: indeed, $(c,d)setminus T$ is a countable disjoint union of intervals $bigcup_n(a_n,b_n)$ and so ${x:a',b'in (a,b)setminus T text{and} b'-a'ge b_n-a_n}$ is a maximal interval in $(a,b)setminus T$.



            Now, either $alpha$ or $betain T$. Suppose $alphain T$. Then, on every interval $(c',c'')$ containing $alpha, f$ is $not$ a polynomial. Choose $c'''$ such that $c'<alpha<c'''<c''<d$. Then, $f$ is not a polynomial on $(c''',c'').$ But since $(c''',c'')subseteq (alpha,beta)$, so we have a contradiction.



            So, either there is no such interval $(c,d)$ or $T$ is empty. In either case, the result follows.






            share|cite|improve this answer











            $endgroup$



            Here is a sketch of a possible approach, which may have some holes, but it is too long for a comment. Maybe someone can patch it up.Picking up on your idea, set



            $T = {tin [a,b]: forall (c,d)ni t: frestriction_{(c,d)}$ is not a polynomial$}$



            Then $T$ is non-empty and closed. By construction, it has no isolated points. Now we may apply the Baire theorem on ${Tcap S_n}$ to find an interval $(c,d)$ such that for some $nin mathbb N, (c,d)cap Tsubset S_n$.



            Now, $f$ is a polynomial on $(c,d)setminus T,$ which is open and so contains an interval $(alpha,beta)$, which we may take to be maximal: indeed, $(c,d)setminus T$ is a countable disjoint union of intervals $bigcup_n(a_n,b_n)$ and so ${x:a',b'in (a,b)setminus T text{and} b'-a'ge b_n-a_n}$ is a maximal interval in $(a,b)setminus T$.



            Now, either $alpha$ or $betain T$. Suppose $alphain T$. Then, on every interval $(c',c'')$ containing $alpha, f$ is $not$ a polynomial. Choose $c'''$ such that $c'<alpha<c'''<c''<d$. Then, $f$ is not a polynomial on $(c''',c'').$ But since $(c''',c'')subseteq (alpha,beta)$, so we have a contradiction.



            So, either there is no such interval $(c,d)$ or $T$ is empty. In either case, the result follows.







            share|cite|improve this answer














            share|cite|improve this answer



            share|cite|improve this answer








            edited Dec 16 '18 at 20:07

























            answered Dec 16 '18 at 17:45









            MatematletaMatematleta

            11.5k2920




            11.5k2920












            • $begingroup$
              Thank you for your answer. I’ll have to think about it for a while.
              $endgroup$
              – Jiu
              Dec 17 '18 at 5:09










            • $begingroup$
              I think my proof needs work and I am hoping somebody can supply the missing details/fix the errors.
              $endgroup$
              – Matematleta
              Dec 17 '18 at 21:12


















            • $begingroup$
              Thank you for your answer. I’ll have to think about it for a while.
              $endgroup$
              – Jiu
              Dec 17 '18 at 5:09










            • $begingroup$
              I think my proof needs work and I am hoping somebody can supply the missing details/fix the errors.
              $endgroup$
              – Matematleta
              Dec 17 '18 at 21:12
















            $begingroup$
            Thank you for your answer. I’ll have to think about it for a while.
            $endgroup$
            – Jiu
            Dec 17 '18 at 5:09




            $begingroup$
            Thank you for your answer. I’ll have to think about it for a while.
            $endgroup$
            – Jiu
            Dec 17 '18 at 5:09












            $begingroup$
            I think my proof needs work and I am hoping somebody can supply the missing details/fix the errors.
            $endgroup$
            – Matematleta
            Dec 17 '18 at 21:12




            $begingroup$
            I think my proof needs work and I am hoping somebody can supply the missing details/fix the errors.
            $endgroup$
            – Matematleta
            Dec 17 '18 at 21:12











            0












            $begingroup$

            You can ‘trace’ the $m$th derivative back to the original function recursively. If $f^{(m)}(x) = 0$ for all $x in [a,b]$, then $f^{(m-1)}(x) = int f^{(m)}(x) , mathrm{d}x = int 0,mathrm{d}x=c_0$. Subsequently, $f^{(m-2)}(x) = int c_0,mathrm{d}x=c_1x+c_0$. Applying this technique $m$ times, we see that $f(x) = c_mx^m + c_{m-1}x^{m-1}+dots + c_1x + c_0$, which is a polynomial by definition.



            (Note that the indices $c_1, c_0$ etc. are not necessarily the same for each iteration, they are just arbitrary constants, where the index indicates the power of $x$ associated to it.)






            share|cite|improve this answer









            $endgroup$













            • $begingroup$
              Thanks for your answer. But the assumption is not that there is an m uniformly for all x, but for all x m-th derivative is zero for m large enough.
              $endgroup$
              – Jiu
              Dec 19 '18 at 13:36










            • $begingroup$
              But doesn't that imply there also exists an $m$ for which $f^{(m)}(x)=0$ for all $x$? (That is, the maximum $m$ that you need for any individual $x$.)
              $endgroup$
              – molarmass
              Dec 19 '18 at 13:40










            • $begingroup$
              the supremum maybe infinite
              $endgroup$
              – Jiu
              Dec 19 '18 at 13:40


















            0












            $begingroup$

            You can ‘trace’ the $m$th derivative back to the original function recursively. If $f^{(m)}(x) = 0$ for all $x in [a,b]$, then $f^{(m-1)}(x) = int f^{(m)}(x) , mathrm{d}x = int 0,mathrm{d}x=c_0$. Subsequently, $f^{(m-2)}(x) = int c_0,mathrm{d}x=c_1x+c_0$. Applying this technique $m$ times, we see that $f(x) = c_mx^m + c_{m-1}x^{m-1}+dots + c_1x + c_0$, which is a polynomial by definition.



            (Note that the indices $c_1, c_0$ etc. are not necessarily the same for each iteration, they are just arbitrary constants, where the index indicates the power of $x$ associated to it.)






            share|cite|improve this answer









            $endgroup$













            • $begingroup$
              Thanks for your answer. But the assumption is not that there is an m uniformly for all x, but for all x m-th derivative is zero for m large enough.
              $endgroup$
              – Jiu
              Dec 19 '18 at 13:36










            • $begingroup$
              But doesn't that imply there also exists an $m$ for which $f^{(m)}(x)=0$ for all $x$? (That is, the maximum $m$ that you need for any individual $x$.)
              $endgroup$
              – molarmass
              Dec 19 '18 at 13:40










            • $begingroup$
              the supremum maybe infinite
              $endgroup$
              – Jiu
              Dec 19 '18 at 13:40
















            0












            0








            0





            $begingroup$

            You can ‘trace’ the $m$th derivative back to the original function recursively. If $f^{(m)}(x) = 0$ for all $x in [a,b]$, then $f^{(m-1)}(x) = int f^{(m)}(x) , mathrm{d}x = int 0,mathrm{d}x=c_0$. Subsequently, $f^{(m-2)}(x) = int c_0,mathrm{d}x=c_1x+c_0$. Applying this technique $m$ times, we see that $f(x) = c_mx^m + c_{m-1}x^{m-1}+dots + c_1x + c_0$, which is a polynomial by definition.



            (Note that the indices $c_1, c_0$ etc. are not necessarily the same for each iteration, they are just arbitrary constants, where the index indicates the power of $x$ associated to it.)






            share|cite|improve this answer









            $endgroup$



            You can ‘trace’ the $m$th derivative back to the original function recursively. If $f^{(m)}(x) = 0$ for all $x in [a,b]$, then $f^{(m-1)}(x) = int f^{(m)}(x) , mathrm{d}x = int 0,mathrm{d}x=c_0$. Subsequently, $f^{(m-2)}(x) = int c_0,mathrm{d}x=c_1x+c_0$. Applying this technique $m$ times, we see that $f(x) = c_mx^m + c_{m-1}x^{m-1}+dots + c_1x + c_0$, which is a polynomial by definition.



            (Note that the indices $c_1, c_0$ etc. are not necessarily the same for each iteration, they are just arbitrary constants, where the index indicates the power of $x$ associated to it.)







            share|cite|improve this answer












            share|cite|improve this answer



            share|cite|improve this answer










            answered Dec 19 '18 at 13:30









            molarmassmolarmass

            1,401615




            1,401615












            • $begingroup$
              Thanks for your answer. But the assumption is not that there is an m uniformly for all x, but for all x m-th derivative is zero for m large enough.
              $endgroup$
              – Jiu
              Dec 19 '18 at 13:36










            • $begingroup$
              But doesn't that imply there also exists an $m$ for which $f^{(m)}(x)=0$ for all $x$? (That is, the maximum $m$ that you need for any individual $x$.)
              $endgroup$
              – molarmass
              Dec 19 '18 at 13:40










            • $begingroup$
              the supremum maybe infinite
              $endgroup$
              – Jiu
              Dec 19 '18 at 13:40




















            • $begingroup$
              Thanks for your answer. But the assumption is not that there is an m uniformly for all x, but for all x m-th derivative is zero for m large enough.
              $endgroup$
              – Jiu
              Dec 19 '18 at 13:36










            • $begingroup$
              But doesn't that imply there also exists an $m$ for which $f^{(m)}(x)=0$ for all $x$? (That is, the maximum $m$ that you need for any individual $x$.)
              $endgroup$
              – molarmass
              Dec 19 '18 at 13:40










            • $begingroup$
              the supremum maybe infinite
              $endgroup$
              – Jiu
              Dec 19 '18 at 13:40


















            $begingroup$
            Thanks for your answer. But the assumption is not that there is an m uniformly for all x, but for all x m-th derivative is zero for m large enough.
            $endgroup$
            – Jiu
            Dec 19 '18 at 13:36




            $begingroup$
            Thanks for your answer. But the assumption is not that there is an m uniformly for all x, but for all x m-th derivative is zero for m large enough.
            $endgroup$
            – Jiu
            Dec 19 '18 at 13:36












            $begingroup$
            But doesn't that imply there also exists an $m$ for which $f^{(m)}(x)=0$ for all $x$? (That is, the maximum $m$ that you need for any individual $x$.)
            $endgroup$
            – molarmass
            Dec 19 '18 at 13:40




            $begingroup$
            But doesn't that imply there also exists an $m$ for which $f^{(m)}(x)=0$ for all $x$? (That is, the maximum $m$ that you need for any individual $x$.)
            $endgroup$
            – molarmass
            Dec 19 '18 at 13:40












            $begingroup$
            the supremum maybe infinite
            $endgroup$
            – Jiu
            Dec 19 '18 at 13:40






            $begingroup$
            the supremum maybe infinite
            $endgroup$
            – Jiu
            Dec 19 '18 at 13:40













            0












            $begingroup$


            b) Show that if a function $fin C^{(infty)}[a,b]$ is such that $forall xin [a,b] ;exists nin mathbb{N} ;forall m>n ;(f^{(m)}(x)=0)$, then $f$ is a polynomial.




            It suffices to require that $f^n(x)=0$ and this is a well-known fact.






            share|cite|improve this answer









            $endgroup$


















              0












              $begingroup$


              b) Show that if a function $fin C^{(infty)}[a,b]$ is such that $forall xin [a,b] ;exists nin mathbb{N} ;forall m>n ;(f^{(m)}(x)=0)$, then $f$ is a polynomial.




              It suffices to require that $f^n(x)=0$ and this is a well-known fact.






              share|cite|improve this answer









              $endgroup$
















                0












                0








                0





                $begingroup$


                b) Show that if a function $fin C^{(infty)}[a,b]$ is such that $forall xin [a,b] ;exists nin mathbb{N} ;forall m>n ;(f^{(m)}(x)=0)$, then $f$ is a polynomial.




                It suffices to require that $f^n(x)=0$ and this is a well-known fact.






                share|cite|improve this answer









                $endgroup$




                b) Show that if a function $fin C^{(infty)}[a,b]$ is such that $forall xin [a,b] ;exists nin mathbb{N} ;forall m>n ;(f^{(m)}(x)=0)$, then $f$ is a polynomial.




                It suffices to require that $f^n(x)=0$ and this is a well-known fact.







                share|cite|improve this answer












                share|cite|improve this answer



                share|cite|improve this answer










                answered Dec 20 '18 at 1:08









                Alex RavskyAlex Ravsky

                42.6k32383




                42.6k32383






























                    draft saved

                    draft discarded




















































                    Thanks for contributing an answer to Mathematics Stack Exchange!


                    • Please be sure to answer the question. Provide details and share your research!

                    But avoid



                    • Asking for help, clarification, or responding to other answers.

                    • Making statements based on opinion; back them up with references or personal experience.


                    Use MathJax to format equations. MathJax reference.


                    To learn more, see our tips on writing great answers.




                    draft saved


                    draft discarded














                    StackExchange.ready(
                    function () {
                    StackExchange.openid.initPostLogin('.new-post-login', 'https%3a%2f%2fmath.stackexchange.com%2fquestions%2f3042725%2fif-at-each-point-of-a-closed-interval-the-m-th-derivative-of-f-is-0-for-m%23new-answer', 'question_page');
                    }
                    );

                    Post as a guest















                    Required, but never shown





















































                    Required, but never shown














                    Required, but never shown












                    Required, but never shown







                    Required, but never shown

































                    Required, but never shown














                    Required, but never shown












                    Required, but never shown







                    Required, but never shown







                    Popular posts from this blog

                    Bundesstraße 106

                    Le Mesnil-Réaume

                    Ida-Boy-Ed-Garten